Docsity
Docsity

Prepare for your exams
Prepare for your exams

Study with the several resources on Docsity


Earn points to download
Earn points to download

Earn points by helping other students or get them with a premium plan


Guidelines and tips
Guidelines and tips

NCLEX EXAM PREPARATIONS QUESTIONS WITH ANSWERS TESTED AND VERIFIED SOLUTONS BEST RATED, Exams of Nursing

NCLEX EXAM PREPARATIONS QUESTIONS WITH ANSWERS TESTED AND VERIFIED SOLUTONS BEST RATED A+ 2024 UPDATES

Typology: Exams

2023/2024

Available from 09/08/2023

hesigrader01
hesigrader01 🇺🇸

3.5

(6)

1.1K documents

1 / 520

Toggle sidebar

Related documents


Partial preview of the text

Download NCLEX EXAM PREPARATIONS QUESTIONS WITH ANSWERS TESTED AND VERIFIED SOLUTONS BEST RATED and more Exams Nursing in PDF only on Docsity! NCLEX EXAM PREPARATIONS QUESTIONS WITH ANSWERS TESTED AND VERIFIED SOLUTONS BEST RATED A+ 2024 UPDATES Airway – Obstruction and Asthma 1. An elderly client with pneumonia may appear with which of the following symptoms first? A. Altered mental status and dehydration B. fever and chills C. Hemoptysis and dyspnea D. Pleuretic chest pain and cough Ans-1. 1. Fever, chills, hemoptysis, dyspnea, cough, and pleuric chest pain are the common symptoms of pneumonia, but elderly clients may first appear with only an altered mental status and dehydration due to a blunted immune response. 2. Which of the following pathophysiological mechanisms that occurs in the lung parenchyma allows pneumonia to develop? 1. Atelectasis 2. Bronchiectasis 3. Effusion 4. Inflammation Ans-2. 4. the most common feature of all types of pneumonia is an inflammatory pulmonary response to the offending organism or agent. Atelectasis and bronchiectasis indicate a collapse of a portion of the airway that doesn’t occur with pneumonia. An effusion is an accumulation of excess pleural fluid in the pleural space, which may be a secondary response to pneumonia. 3. A 7-year-old client is brought to the E.R. He’s tachypneic and afebrile and has a respiratory rate of 36 breaths/minute and a nonproductive cough. He recently had a cold. From his history, the client may have which of the following? 1. Acute asthma 2. Bronchial pneumonia 3. Chronic obstructive pulmonary disease (COPD) 4. Emphysema Ans-3. 1. Based on the client’s history and symptoms, acute asthma is the most likely diagnosis. He’s unlikely to have bronchial pneumonia without a productive cough and fever and he’s too young to have developed COPD or emphysema. 4. Which of the following assessment findings would help confirm a diagnosis of asthma in a client suspected of having the disorder? 1. Circumoral cyanosis 2. Increased forced expiratory volume 3. Inspiratory and expiratory wheezing 4. Normal breath sounds Ans-4. 3. Inspiratory and expiratory wheezes are typical findings in asthma. Circumoral cyanosis may be present in extreme cases of respiratory distress. The nurse would expect the client to have a decreased forced expiratory volume because asthma is an obstructive pulmonary disease. Breath sounds will be “tight” sounding or markedly decreased; they won’t be normal. 5. Which of the following types of asthma involves an acute asthma attack brought on by an upper respiratory infection? 1. Emotional 2. Extrinsic 3. Intrinsic 4. Mediated Ans-5. 1. Intrinsic asthma doesn’t have an easily identifiable allergen and can be triggered by the common cold. Asthma caused be emotional reasons is considered to be in the extrinsic category. Extrinsic asthma is caused by dust, molds, and pets; easily identifiable allergens. Mediated asthma doesn’t exist. 6. A client with acute asthma showing inspiratory and expiratory wheezes and a decreased expiratory volume should be treated with which of the following classes of medication right away? 1. Beta-adrenergic blockers 2. Bronchodilators 4. Emphysema Ans-11. 4. These are classic signs and symptoms of a client with emphysema. Clients with ARDS are acutely short of breath and require emergency care; those with asthma are also acutely short of breath during an attack and appear very frightened. Clients with chronic obstructive bronchitis are bloated and cyanotic in appearance. 12. Its highly recommended that clients with asthma, chronic bronchitis, and emphysema have Pneumovax and flu vaccinations for which of the following reasons? 1. All clients are recommended to have these vaccines 2. These vaccines produce bronchodilation and improve oxygenation. 3. These vaccines help reduce the tachypnea these clients experience. 4. Respiratory infections can cause severe hypoxia and possibly death in these clients. Ans-12. 4. it’s highly recommended that clients with respiratory disorders be given vaccines to protect against respiratory infection. Infections can cause these clients to need intubation and mechanical ventilation, and it may be difficult to wean these clients from the ventilator. The vaccines have no effect on bronchodilation or respiratory care. 13. Exercise has which of the following effects on clients with asthma, chronic bronchitis, and emphysema? 1. It enhances cardiovascular fitness. 2. It improves respiratory muscle strength. 3. It reduces the number of acute attacks. 4. It worsens respiratory function and is discouraged. Ans-13. 1. Exercise can improve cardiovascular fitness and help the client tolerate periods of hypoxia better, perhaps reducing the risk of heart attack. Most exercise has little effect on respiratory muscle strength, and these clients can’t tolerate the type of exercise necessary to do this. Exercise won’t reduce the number of acute attacks. In some instances, exercise may be contraindicated, and the client should check with his physician before starting any exercise program. 14. Clients with chronic obstructive bronchitis are given diuretic therapy. Which of the following reasons best explains why? 1. Reducing fluid volume reduces oxygen demand. 2. Reducing fluid volume improves clients’ mobility. 3. Restricting fluid volume reduces sputum production. 4. Reducing fluid volume improves respiratory function. Ans-14. 1. Reducing fluid volume reduces the workload of the heart, which reduces oxygen demand and, in turn, reduces the respiratory rate. It may also reduce edema and improve mobility a little, but exercise tolerance will still be harder to clear airways. Reducing fluid volume won’t improve respiratory function, but may improve oxygenation. 15. A 69-year-old client appears thin and cachectic. He’s short of breath at rest and his dyspnea increases with the slightest exertion. His breath sounds are diminished even with deep inspiration. These signs and symptoms fit which of the following conditions? 1. ARDS 2. Asthma 3. Chronic obstructive bronchitis 4. Emphysema Ans-15. 4. In emphysema, the wall integrity of the individual air sacs is damaged, reducing the surface area available for gas exchange. Very little air movement occurs in the lungs because of bronchiole collapse, as well. In ARDS, the client’s condition is more acute and typically requires mechanical ventilation. In asthma and bronchitis, wheezing is prevalent. 16. A client with emphysema should receive only 1 to 3 L/minute of oxygen, if needed, or he may lose his hypoxic drive. Which of the following statements is correct about hypoxic drive? 1. The client doesn’t notice he needs to breathe. 2. The client breathes only when his oxygen levels climb above a certain point. 3. The client breathes only when his oxygen levels dip below a certain point. 4. The client breathes only when his carbon dioxide level dips below a certain point. Ans-16. 3. Clients with emphysema breathe when their oxygen levels drop to a certain level; this is known as the hypoxic drive. They don’t take a breath when their levels of carbon dioxide are higher than normal, as do those with healthy respiratory physiology. If too much oxygen is given, the client has little stimulus to take another breath. In the meantime, his carbon dioxide levels continue to climb, and the client will pass out, leading to a respiratory arrest. 17. Teaching for a client with chronic obstructive pulmonary disease (COPD) should include which of the following topics? 1. How to have his wife learn to listen to his lungs with a stethoscope from Wal-Mart. 2. How to increase his oxygen therapy. 3. How to treat respiratory infections without going to the physician. 4. How to recognize the signs of an impending respiratory infection. Ans-17. 4. Respiratory infection in clients with a respiratory disorder can be fatal. It’s important that the client understands how to recognize the signs and symptoms of an impending respiratory infection. It isn’t appropriate for the wife to listen to his lung sounds, besides, you can’t purchase stethoscopes from Wal-Mart. If the client has signs and symptoms of an infection, he should contact his physician at once. 18. Which of the following respiratory disorders is most common in the first 24 to 48 hours after surgery? 1. Atelectasis 2. Bronchitis 3. Pneumonia 4. Pneumothorax Ans-18. 1. Atelectasis develops when there’s interference with the normal negative pressure that promotes lung expansion. Clients in the postoperative phase often splint their breathing because of pain and positioning, which causes hypoxia. It’s uncommon for any of the other respiratory disorders to develop. 19. Which of the following measures can reduce or prevent the incidence of atelectasis in a post-operative client? Ans-24. 4. A client about to go into respiratory arrest will have inefficient ventilation and will be retaining carbon dioxide. The value expected would be around 80 mm Hg. All other values are lower than expected. 25. A client has started a new drug for hypertension. Thirty minutes after he takes the drug, he develops chest tightness and becomes short of breath and tachypneic. He has a decreased level of consciousness. These signs indicate which of the following conditions? 1. Asthma attack 2. Pulmonary embolism 3. respiratory failure 4. Rheumatoid arthritis Ans-25. 3. The client was reacting to the drug with respiratory signs of impending anaphylaxis, which could lead to eventual respiratory failure. Although the signs are also related to an asthma attack or a pulmonary embolism, consider the new drug first. Rheumatoid arthritis doesn’t manifest these signs. 26. Emergency treatment for a client with impending anaphylaxis secondary to hypersensitivity to a drug should include which of the following actions first? 1. Administering oxygen 2. Inserting an I.V. catheter 3. Obtaining a complete blood count (CBC) 4. Taking vital signs Ans-26. 1. Giving oxygen would be the best first action in this case. Vital signs then should be checked and the physician immediately notified. If the client doesn’t already have an I.V. catheter, one may be inserted now if anaphylactic shock is developing. Obtaining a CBC wouldn’t help the emergency situation. 27. Following the initial care of a client with asthma and impending anaphylaxis from hypersensitivity to a drug, the nurse should take which of the following steps next? 1. Administer beta-adrenergic blockers 2. Administer bronchodilators 3 3. Obtain serum electrolyte levels 4. Have the client lie flat in the bed. Ans-27. 2. Bronchodilators would help open the client’s airway and improve his oxygenation status. Beta-adrenergic blockers aren’t indicated in the management of asthma because they may cause bronchospasm. Obtaining laboratory values wouldn’t be done on an emergency basis, and having the client lie flat in bed could worsen his ability to breathe. 28. A client’s ABG results are as follows: pH: 7.16; PaCO2 80 mm Hg; PaO2 46 mm Hg; HCO – 24 mEq/L; SaO2 81%. This ABG result represents which of the following conditions? 1. Metabolic acidosis 2. Metabolic alkalosis 3. Respiratory acidosis 4. Respiratory alkalosis Ans-28. 3. You all should know this. Practice some problems if you got this wrong. 29. A nurse plans care for a client with chronic obstructive pulmonary disease, knowing that the client is most likely to experience what type of acid-base imbalance? 1. Respiratory acidosis 2. Respiratory alkalosis 3. Metabolic acidosis 4. Metabolic alkalosis Ans-29. 1. Respiratory acidosis is most often due to hypoventilation. Chronic respiratory acidosis is most commonly caused by COPD. In end-stage disease, pathological changes lead to airway collapse, air trapping, and disturbance of ventilation-perfusion relationships. 30. A nurse is caring for a client who is on a mechanical ventilator. Blood gas results indicate a pH of 7.50 and a PCO2 of 30 mm Hg. The nurse has determined that the client is experiencing respiratory alkalosis. Which laboratory value would most likely be noted in this condition? 1. Sodium level of 145 mEq/L 2. Potassium level of 3.0 mEq/L 3. Magnesium level of 2.0 mg/L 4. Phosphorus level of 4.0 mg/dl Ans-30. 2. Clinical manifestations of respiratory alkalosis include headache, tachypnea, paresthesias, tetany, vertigo, convulsions, hypokalemia, and hypocalcemia. Options 1, 3, and 4 identify normal laboratory values. Option 2 identifies the presence of hypokalemia. 31. A nurse reviews the arterial blood gas results of a patient and notes the following: pH 7.45; PCO2 30 mm Hg; and bicarbonate concentration of 22 mEq/L. The nurse analyzes these results as indicating: 1. Metabolic acidosis, compensated. 2. Metabolic alkalosis, uncompensated. 3. Respiratory alkalosis, compensated. 4. Respiratory acidosis, compensated. Ans-31. 3. The normal pH is 7.35 to 7.45. In a respiratory condition, an opposite (see-saw) will be seen between the pH and the PCO2. In this situation, the pH is at the high end of the normal value and the PCO2 is low. In an alkalotic condition, the pH is up. Therefore, the values identified in the question indicate a respiratory alkalosis. Compensation occurs when the pH returns to a normal value. Because the pH is in the normal range at the high end, compensation has occurred. 32. A client is scheduled for blood to be drawn from the radial artery for an ABG determination. Before the blood is drawn, an Allen’s test is performed to determine the adequacy of the: 1. Popliteal circulation 2. Ulnar circulation 3. Femoral circulation 4. Carotid circulation Ans-32. 2. Before radial puncture for obtaining an ABG, you should perform an Allen’s test to determine adequate ulnar circulation. Failure to determine the presence of adequate collateral circulation could result in severe ischemic injury o the hand if damage to the radial artery occurs with arterial puncture. 5. Explain the procedure to the client. Ans-38. 5, 2, 1, 4, and then 3. 39. A nurse is preparing to obtain a sputum specimen from a client. Which of the following nursing actions will facilitate obtaining the specimen? 1. Limiting fluids 2. Having the client take 3 deep breaths. 3. Asking the client to spit into the collection container. 4. Asking the client to obtain the specimen after eating. Ans-39. 2. To obtain a sputum specimen, the client should rinse the mouth to prevent contamination, breathe deeply, and then cough unto a sputum specimen container. The client should be encouraged to cough and not spit so as to obtain sputum. Sputum can be thinned by fluids or by a respiratory treatment such as inhalation of nebulized saline or water. The optimal time to obtain a specimen is on arising in the morning. 40. A nurse is caring for a client after a bronchoscopy and biopsy. Which of the following signs if noted in the client should be reported immediately to the physician? 1. Blood-streaked sputum 2. Dry cough 3. Hematuria 4. Bronchospasm Ans-40. 4. If a biopsy was performed during a bronchoscopy, blood streaked sputum is expected for several hours. Frank blood indicates hemorrhage. A dry cough may be expected. The client should be assessed for signs of complications, which would include cyanosis, dyspnea, stridor, bronchospasm, hemoptysis, hypotension, tachycardia, and dysrhythmias. Hematuria is unrelated to this procedure. 41. A nurse is suctioning fluids from a client via a tracheostomy tube. When suctioning, the nurse must limit the suctioning to a maximum of: 1. 5 seconds 2. 10 seconds 3. 30 seconds 4. 1 minute Ans-41. 2. Hypoxemia can be caused by prolonged suctioning, which stimulates the pacemaker cells within the heart. A vasovagal response may occur causing bradycardia. The nurse must preoxygenate the client before suctioning and limit the suctioning pass to 10 seconds. 42. A nurse is suctioning fluids from a client through an endotracheal tube. During the suctioning procedure, the nurse notes on the monitor that the heart rate decreases. Which of the following is the most appropriate nursing intervention? 1. Continue to suction 2. Ensure that the suction is limited to 15 seconds 3. Stop the procedure and reoxyenate the client 4. Notify the physician immediately. Ans-42. 3. During suctioning, the nurse should monitor the client closely for side effects, including hypoxemia, cardiac irregularities such as a decrease in HR resulting from vagal stimulation, mucosal trauma, hypotension, and paroxysmal coughing. If side effects develop, especially cardiac irregularities, this procedure is stopped and the client is reoxygenated. 43. An unconscious client is admitted to an emergency room. Arterial blood gas measurements reveal a pH of 7.30, a low bicarbonate level, a normal carbon dioxide level, and a normal oxygen level. An elevated potassium level is also present. These results indicate the presence of: 1. Metabolic acidosis 2. Respiratory acidosis 3. Combined respiratory and metabolic acidosis 4. overcompensated respiratory acidosis Ans-43. 1. In an acidotic condition the pH would be low, indicating the acidosis. In addition, a low bicarbonate level along with the pH would indicate a metabolic state. 44. A nurse is caring for a client hospitalized with acute exacerbation of COPD. Which of the following would the nurse expect to note on assessment of this client? 1. Increased oxygen saturation with exercise 2. Hypocapnia 3. A hyperinflated chest on x-ray film 4. A widened diaphragm noted on chest x-ray film Ans-44. 3. Clinical manifestations of COPD include hypoxemia, hypercapnia, dyspnea on exertion and at rest, oxygen desaturation with exercise, and the use of accessory muscles of respiration. Chest x-ray films reveal a hyperinflated chest and a flattened diaphragm is the disease is advanced. 45. An oxygenated delivery system is prescribed for a client with COPD to deliver a precise oxygen concentration. Which of the following types of oxygen delivery systems would the nurse anticipate to be prescribed? 1. Venturi mask 2. Aerosol mask 3. Face tent 4. Tracheostomy collar Ans-45. 1. The venture mask delivers the most accurate oxygen concentration. The Venturi mask is the best oxygen delivery system for the client with chronic airflow limitation because it delivers a precise oxygen concentration. The face tent, the aerosol mask, and the tracheostomy collar are also high-flow oxygen delivery systems but most often are used to administer high humidity. 46. Theophylline (Theo-Dur) tablets are prescribed for a client with chronic airflow limitation, and the nurse instructs the client about the medication. Which statement by the client indicates a need for further teaching? Ans-52. 4. Side effects that can occur from a beta 2 agonist include tremors, nausea, nervousness, palpitations, tachycardia, peripheral vasodilation, and dryness of the mouth or throat. 53. A nurse teaches a client about the use of a respiratory inhaler. Which action by the client indicated a need for further teaching? 1. Removes the cap and shakes the inhaler well before use. 2. Presses the canister down with finger as he breathes in. 3. Inhales the mist and quickly exhales. 4. Waits 1 to 2 minutes between puffs if more than one puff has been prescribed. Ans-53. 3. The client should be instructed to hold his or her breath at least 10 to 15 seconds before exhaling the mist. 54. A female client is scheduled to have a chest radiograph. Which of the following questions is of most importance to the nurse assessing this client? 1. “Is there any possibility that you could be pregnant?” 2. “Are you wearing any metal chains or jewelry?” 3. “Can you hold your breath easily?” 4. “Are you able to hold your arms above your head?” Ans-54. 1. the most important item to ask about is the client’s pregnancy status because pregnant women should not be exposed to radiation. Clients are also asked to remove any chains or metal objects that could interfere with obtaining an adequate film. A chest radiograph most often is done at full inspiration, which gives optimal lung expansion. If a lateral view of the chest is ordered, the client is asked to raise the arms above the head. Most films are done in posterior- anterior view. 55. A client has just returned to a nursing unit following bronchoscopy. A nurse would implement which of the following nursing interventions for this client? 1. Encouraging additional fluids for the next 24 hours 2. Ensuring the return of the gag reflex before offering foods or fluids 3. Administering atropine intravenously 4. Administering small doses of midazolam (Versed). Ans-55. 2. After bronchoscopy, the nurse keeps the client on NPO status until the gag reflex returns because the preoperative sedation and the local anesthesia impair swallowing and the protective laryngeal reflexes for a number of hours. Additional fluids is unnecessary because no contrast dye is used that would need to be flushed from the system. Atropine and Versed would be administered before the procedure, not after. 56. A client has an order to have radial ABG drawn. Before drawing the sample, a nurse occludes the: 1. Brachial and radial arteries, and then releases them and observes the circulation of the hand. 2. Radial and ulnar arteries, releases one, evaluates the color of the hand, and repeats the process with the other artery. 3. Radial artery and observes for color changes in the affected hand. 4. Ulnar artery and observes for color changes in the affected hand. Ans-56. 2. Before drawing an ABG, the nurse assesses the collateral circulation to the hand with Allen’s test. This involves compressing the radial and ulnar arteries and asking the client to close and open the fist. This should cause the hand to become pale. The nurse then releases pressure on one artery and observes whether circulation is restored quickly. The nurse repeats the process, releasing the other artery. The blood sample may be taken safely if collateral circulation is adequate. 57. A nurse is assessing a client with chronic airflow limitation and notes that the client has a “barrel chest.” The nurse interprets that this client has which of the following forms of chronic airflow limitation? 1. Chronic obstructive bronchitis 2. Emphysema 3. Bronchial asthma 4. Bronchial asthma and bronchitis Ans-57. 2. The client with emphysema has hyperinflation of the alveoli and flattening of the diaphragm. These lead to increased anteroposterior diameter, which is referred to as “barrel chest.” The client also has dyspnea with prolonged expiration and has hyperresonant lungs to percussion. 58. A client has been taking benzonatate (Tessalin Perles) as prescribed. A nurse concludes that the medication is having the intended effect if the client experiences: 1. Decreased anxiety level 2. Increased comfort level 3. Reduction of N/V 4. Decreased frequency and intensity of cough Ans-58. 4. Benzonatate is a locally acting antitussive the effectiveness of which is measured by the degree to which it decreases the intensity and frequency of cough without eliminating the cough reflex 59. Which of the following would be an expected outcome for a client recovering from an upper respiratory tract infection? The client will: 1. Maintain a fluid intake of 800ml every 24 hours. 2. Experience chills only once a day 3. Cough productively without chest discomfort. 4. Experience less nasal obstruction and discharge. Ans-59. 4. A client recovering from an URI should report decreasing or no nasal discharge and obstruction. Daily fluid intake should be increase to more than 1 L every 24 hours to liquefy secretions. The temperature should be below 100*F (37.8*C) with no chills or diaphoresis. A productive cough with chest pain indicated pulmonary infection, not an URI 60. Which of the following individuals would the nurse consider to have the highest priority for receiving an influenza vaccination? 1. A 60-year-old man with a hiatal hernia 2. A 36-year-old woman with 3 children 3. A 50-year-old woman caring for a spouse with cancer Ans-65. 1. The client’s problem is altered nutrition—specifically, less than required. The cause, as stated by the client, is the fatigue associated with the disease process. Activity intolerance is a likely diagnosis but is not related to the client’s nutritional problems. Weight loss is not a nursing diagnosis. Ineffective breathing pattern may be a problem, but this diagnosis does not specifically address the problem of weight loss described by the client. 66. When developing a discharge plan to manage the care of a client with COPD, the nurse should anticipate that the client will do which of the following? 1. Develop infections easily 2. Maintain current status 3. Require less supplemental oxygen 4. Show permanent improvement. Ans-66. 1. A client with COPD is at high risk for development of respiratory infections. COPD is a slowly progressive; therefore, maintaining current status and establishing a goal that the client will require less supplemental oxygen are unrealistic expectations. Treatment may slow progression of the disease, but permanent improvement is highly unlikely 67. Which of the following outcomes would be appropriate for a client with COPD who has been discharged to home? The client: 1. Promises to do pursed lip breathing at home. 2. States actions to reduce pain. 3. States that he will use oxygen via a nasal cannula at 5 L/minute. 4. Agrees to call the physician if dyspnea on exertion increases. Ans-67. 4. Increasing dyspnea on exertion indicates that the client may be experiencing complications of COPD, and therefore the physician should be notified. Extracting promises from clients is not an outcome criterion. Pain is not a common symptom of COPD. Clients with COPD use low-flow oxygen supplementation (1 to 2 L/minute) to avoid suppressing the respiratory drive, which, for these clients, is stimulated by hypoxia. 68. Which of the following physical assessment findings would the nurse expect to find in a client with advanced COPD? 1. Increased anteroposterior chest diameter 2. Underdeveloped neck muscles 3. Collapsed neck veins 4. Increased chest excursions with respiration Ans-68. 1. Increased anteroposterior chest diameter is characteristic of advanced COPD. Air is trapped in the overextended alveoli, and the ribs are fixed in an inspiratory position. The result is the typical barrel-chested appearance. Overly developed, not underdeveloped, neck muscles are associated with COPD because of their increased use in the work of breathing. Distended, not collapsed, neck veins are associated with COPD as a symptom of the heart failure that the client may experience secondary to the increased workload on the heart to pump into pulmonary vasculature. Diminished, not increased, chest excursion is associated with COPD. 69. Which of the following is the primary reason to teach pursed-lip breathing to clients with emphysema? 1. To promote oxygen intake 2. To strengthen the diaphragm 3. To strengthen the intercostal muscles 4. To promote carbon dioxide elimination Ans-69. 4. Pursed lip breathing prolongs exhalation and prevents air trapping in the alveoli, thereby promoting carbon dioxide elimination. By prolonged exhalation and helping the client relax, pursed-lip breathing helps the client learn to control the rate and depth of respiration. Pursed-lip breathing does not promote the intake of oxygen, strengthen the diaphragm, or strengthen intercostal muscles. 70. Which of the following is a priority goal for the client with COPD? 1. Maintaining functional ability 2. Minimizing chest pain 3. Increasing carbon dioxide levels in the blood 4. Treating infectious agents 3 Ans-70. 1. A priority goal for the client with COPD is to manage the s/s of the disease process so as to maintain the client’s functional ability. Chest pain is not a typical sign of COPD. The carbon dioxide concentration in the blood is increased to an abnormal level in clients with COPD; it would not be a goal to increase the level further. Preventing infection would be a goal of care for the client with COPD. 71. A client’s arterial blood gas levels are as follows: pH 7.31; PaO2 80 mm Hg, PaCO2 65 mm Hg; HCO – 36 mEq/L. Which of the following signs or symptoms would the nurse expect? 1. Cyanosis 2. Flushed skin 3. Irritability 4. Anxiety Ans-71. 2. The high PaCO2 level causes flushing due to vasodilation. The client also becomes drowsy and lethargic because carbon dioxide has a depressant effect on the CNS. Cyanosis is a late sign of hypoxia. Irritability and anxiety are not common with a PaCO2 level of 65 mm Hg but are associated with hypoxia. 72. When teaching a client with COPD to conserve energy, the nurse should teach the client to lift objects: 1. While inhaling through an open mouth. 2. While exhaling through pursed lips 3. After exhaling but before inhaling. 4. While taking a deep breath and holding it. Ans-72. 2. Exhaling requires less energy than inhaling. Therefore, lifting while exhaling saves energy and reduced perceived dyspnea. Pursing the lips prolongs exhalation and provides the client with more control over breathing. Lifting after exhalation but before inhaling is similar to lifting with the breath held. This should not be recommended because it is similar to the Valsalva maneuver, which can stimulate cardiac dysrhythmias. 73. The nurse teaches a client with COPD to assess for s/s of right-sided heart failure. Which of the following s/s would be included in the teaching plan? 1. Clubbing of nail beds 78. A 34-year-old woman with a history of asthma is admitted to the emergency department. The nurse notes that the client is dyspneic, with a respiratory rate of 35 breaths/minute, nasal flaring, and use of accessory muscles. Auscultation of the lung fields reveals greatly diminished breath sounds. Based on these findings, what action should the nurse take to initiate care of the client? 1. Initiate oxygen therapy and reassess the client in 10 minutes. 2. Draw blood for an ABG analysis and send the client for a chest x-ray. 3. Encourage the client to relax and breathe slowly through the mouth 4. Administer bronchodilators Ans-78. 4. In an acute asthma attack, diminished or absent breath sounds can be an ominous sign of indicating lack of air movement in the lungs and impending respiratory failure. The client requires immediate intervention with inhaled bronchodilators, intravenous corticosteroids, and possibly intravenous theophylline. Administering oxygen and reassessing the client 10 minutes later would delay needed medical intervention, as would drawing an ABG and obtaining a chest x-ray. It would be futile to encourage the client to relax and breathe slowly without providing necessary pharmacologic intervention. 79. The nurse would anticipate which of the following ABG results in a client experiencing a prolonged, severe asthma attack? 1. Decreased PaCO2, increased PaO2, and decreased pH. 2. Increased PaCO2, decreased PaO2, and decreased pH. 3. Increased PaCO2, increased PaO2, and increased pH. 4. Decreased PaCO2, decreased PaO2, and increased pH. Ans-79. 2. As the severe asthma attack worsens, the client becomes fatigued and alveolar hypotension develops. This leads to carbon dioxide retention and hypoxemia. The client develops respiratory acidosis. Therefore, the PaCO2 level increase, the PaO2 level decreases, and the pH decreases, indicating acidosis. 80. A client with acute asthma is prescribed short-term corticosteroid therapy. What is the rationale for the use of steroids in clients with asthma? 1. Corticosteroids promote bronchodilation 2. Corticosteroids act as an expectorant 3. Corticosteroids have an anti-inflammatory effect 4. Corticosteroids prevent development of respiratory infections. Ans-80. 3. Corticosteroids have an anti-inflammatory effect and act to decrease edema in the bronchial airways and decrease mucus secretion. Corticosteroids do not have a bronchodilator effect, act as expectorants, or prevent respiratory infections. 81. The nurse is teaching the client how to use a metered dose inhaler (MDI) to administer a Corticosteroid drug. Which of the following client actions indicates that he us using the MDI correctly? Select all that apply. 1. The inhaler is held upright. 2. Head is tilted down while inhaling the medication 3. Client waits 5 minutes between puffs. 4. Mouth is rinsed with water following administration 5. Client lies supine for 15 minutes following administration. Ans-81. 1 and 4. 82. A client is prescribed metaproterenol (Alupent) via a metered dose inhaler (MDI), two puffs every 4 hours. The nurse instructs the client to report side effects. Which of the following are potential side effects of metaproterenol? 1. Irregular heartbeat 2. Constipation 3. Petal edema 4. Decreased heart rate. Ans-82. 1. Irregular heart rates should be reported promptly to the care provider. Metaproterenol may cause irregular heartbeat, tachycardia, or anginal pain because of its adrenergic effect on the beta-adrenergic receptors in the heart. It is not recommended for use in clients with known cardiac disorders. Metaproterenol does not cause constipation, petal edema, or bradycardia. 83. A client has been taking flunisolide (Aerobid), two inhalations a day, for treatment of asthma. He tells the nurse that he has painful, white patches in his mouth. Which response by the nurse would be the most appropriate? 1. “This is an anticipated side-effect of your medication. It should go away in a couple of weeks.” 2. “You are using your inhaler too much and it has irritated your mouth.” 3. “You have developed a fungal infection from your medication. It will need to be treated with an antibiotic.” 4. “Be sure to brush your teeth and floss daily. Good oral hygiene will treat this problem.” Ans-83. 3. Use of oral inhalant corticosteroids, such as flunisolide, can lead to the development of oral thrush, a fungal infection. Once developed, thrush must be treated by antibiotic therapy; it will not resolve on its own. Fungal infections can develop even without overuse of the Corticosteroid inhaler. Although good oral hygiene can help prevent development of a fungal infection, it cannot be used alone to treat the problem. 84. Which of the following health promotion activities should the nurse include in the discharge teaching plan for a client with asthma? 1. Incorporate physical exercise as tolerated into the treatment plan. 2. Monitor peak flow numbers after meals and at bedtime. 3. Eliminate stressors in the work and home environment 4. Use sedatives to ensure uninterrupted sleep at night. 4. Hypotension and hypothermia Ans-89. 3. Increased pulse and pallor are symptoms associated with shock. A compromised venous return may occur if there is a mediastinal shift as a result of excessive fluid removal. Usually no more than 1 L of fluid is removed at one time to prevent this from occurring. 90. If a client continues to hypoventilate, the nurse will continually assess for a complication of: 1. Respiratory acidosis 2. Respiratory alkalosis 3. Metabolic acidosis 4. Metabolic alkalosis Ans-90. 1. Respiratory acidosis represents an increase in the acid component, carbon dioxide, and an increase in the hydrogen ion concentration (decreased pH) of the arterial blood. 91. A client is admitted to the hospital with acute bronchitis. While taking the client’s VS, the nurse notices he has an irregular pulse. The nurse understands that cardiac arrhythmias in chronic respiratory distress are usually the result of: 1. Respiratory acidosis 2. A build-up of carbon dioxide 3. A build-up of oxygen without adequate expelling of carbon dioxide. 4. An acute respiratory infection. Ans-91. 2. The arrhythmias are caused by a build-up of carbon dioxide and not enough oxygen so that the heart is in a constant state of hypoxia 92. Auscultation of a client’s lungs reveals crackles in the left posterior base. The nursing intervention is to: 1. Repeat auscultation after asking the client to deep breathe and cough. 2. Instruct the client to limit fluid intake to less than 2000 ml/day. 3. Inspect the client’s ankles and sacrum for the presence of edema 4. Place the client on bed rest in a semi-Fowlers position. Ans-92. 1. Although crackles often indicate fluid in the alveoli, they may also be related to hypoventilation and will clear after a deep breath or a cough. It is, therefore, premature to impose fluid (2) or activity (4) restrictions (which Margaret would totally do if Dani weren’t there to smack her). Inspection for edema (3) would be appropriate after reauscultation. 93. The most reliable index to determine the respiratory status of a client is to: 1. Observe the chest rising and falling 2. Observe the skin and mucous membrane color. 3. Listen and feel the air movement. 4. Determine the presence of a femoral pulse. Ans-93. 3. To check for breathing, the nurse places her ear and cheek next to the client’s mouth and nose to listen and feel for air movement. The chest rising and falling (1) is not conclusive of a patent airway. Observing skin color (2) is not an accurate assessment of respiratory status, nor is checking the femoral pulse. 94. A client with COPD has developed secondary polycythemia. Which nursing diagnosis would be included in the plan of care because of the polycythemia? 1. Fluid volume deficit related to blood loss. 2. Impaired tissue perfusion related to thrombosis 3. Activity intolerance related to dyspnea 4. Risk for infection related to suppressed immune response. Ans-94. 2. Chronic hypoxia associated with COPD may stimulate excessive RBC production (polycythemia). This results in increased blood viscosity and the risk of thrombosis. The other nursing diagnoses are not applicable in this situation. 95. The physician has scheduled a client for a left pneumonectomy. The position that will most likely be ordered postoperatively for his is the: 1. Unoperative side or back 2. Operative side or back 3. Back only 4. Back or either side. Ans-95. 2. Positioning the client on the operative side facilitates the accumulation of serisanguineous fluid. The fluid forms a solid mass, which prevents the remaining lung from being drawn into the space. 96. Assessing a client who has developed atelectasis postoperatively, the nurse will most likely find: 1. A flushed face 2. Dyspnea and pain 3. Decreased temperature 4. Severe cough and no pain. Ans-96. 2. Atelectasis is a collapse of the alveoli due to obstruction or hypoventilation. Clients become short of breath, have a high temperature, and usually experience severe pain but do not have a severe cough (4). The shortness of breath is a result of decreased oxygen-carbon dioxide exchange at the alveolar level. 97. A fifty-year-old client has a tracheostomy and requires tracheal suctioning. The first intervention in completing this procedure would be to: 1. Change the tracheostomy dressing 2. Provide humidity with a trach mask 3. Apply oral or nasal suction 4. Deflate the tracheal cuff Ans-97. 3. Before deflating the tracheal cuff (4), the nurse will apply oral or nasal suction to the airway to prevent secretions from falling into the lung. Dressing change (1) and humidity (2) do not relate to suctioning. Ans-4. 1. Fever, chills, hemoptysis, dyspnea, cough, and pleuritic chest pain are common symptoms of pneumonia, but elderly clients may first appear with only an altered mental status and dehydration due to a blunted immune response. 5. When auscultating the chest of a client with pneumonia, the nurse would expect to hear which of the following sounds over areas of consolidation? 1. Bronchial 2. Bronchovestibular 3. Tubular 4. Vesicular Ans-5. 1. Chest auscultation reveals bronchial breath sounds over areas of consolidation. Bronchiovesicular are normal over mid-lobe lung regions, tubular sounds are commonly heard over large airways, and vesicular breath sounds are commonly heard in the bases of the lung fields. 6. A diagnosis of pneumonia is typically achieved by which of the following diagnostic tests? 1. ABG analysis 2. Chest x-ray 3. Blood cultures 4. sputum culture and sensitivity Ans-6. 4. Sputum C & S is the best way to identify the organism causing the pneumonia. Chest x-ray will show the area of lung consolidation. ABG analysis will determine the extent of hypoxia present due to the pneumonia, and blood cultures will help determine if the infection is systemic. 7. A client with pneumonia develops dyspnea with a respiratory rate of 32 breaths/minute and difficulty expelling his secretions. The nurse auscultates his lung fields and hears bronchial sounds in the left lower lobe. The nurse determines that the client requires which of the following treatments first? 1. Antibiotics 2. Bed rest 3. Oxygen 4. Nutritional intake Ans-7. 3. The client is having difficulty breathing and is probably becoming hypoxic. As an emergency measure, the nurse can provide oxygen without waiting for a physician’s order. Antibiotics may be warranted, but this isn’t a nursing decision. The client should be maintained on bed rest if he is dyspneic to minimize his oxygen demands, but providing additional will deal more immediately with his problem. The client will need nutritional support, but while dyspneic, he may be unable to spare the energy needed to eat and at the same time maintain adequate oxygenation. 8. A client has been treated with antibiotic therapy for right lower-lobe pneumonia for 10 days and will be discharged today. Which of the following physical findings would lead the nurse to believe it is appropriate to discharge this client? 1. Continued dyspnea 2. Fever of 102*F 3. Respiratory rate of 32 breaths/minute 4. Vesicular breath sounds in right base Ans-8. 4. If the client still has pneumonia, the breath sounds in the right base will be bronchial, not the normal vesicular breath sounds. If the client still has dyspnea, fever, and increased respiratory rate, he should be examined by the physician before discharge because he may have another source of infection or still have pneumonia. 9. The right forearm of a client who had a purified protein derivative (PPD) test for tuberculosis is reddened and raised about 3mm where the test was given. This PPD would be read as having which of the following results? 1. Indeterminate 2. Needs to be redone 3. Negative 4. Positive Ans-9. 3. This test would be classed as negative. A 5mm raised area would be a positive result if a client was HIV+ or had recent close contact with someone diagnosed with TB. Indeterminate isn’t a term used to describe results of a PPD test. If the PPD is reddened and raised 10mm or more, it’s considered positive according to the CDC. 10. A client with primary TB infection can expect to develop which of the following conditions? 1. Active TB within 2 weeks 2. Active TB within 1 month 3. A fever that requires hospitalization 4. A positive skin test Ans-10. 4. A primary TB infection occurs when the bacillus has successfully invaded the entire body after entering through the lungs. At this point, the bacilli are walled off and skin tests read positive. However, all but infants and immunosuppressed people will remain asymptomatic. The general population has a 10% risk of developing active TB over their lifetime, in many cases because of a break in the body’s immune defenses. The active stage shows the classic symptoms of TB: fever, hemoptysis, and night sweats. 11. A client was infected with TB 10 years ago but never developed the disease. He’s now being treated for cancer. The client begins to develop signs of TB. This is known as which of the following types of infection? 1. Active infection 2. Primary infection 3. Superinfection 4. Tertiary infection Ans-11. 1. Some people carry dormant TB infections that may develop into active disease. In addition, primary sites of infection containing TB bacilli may remain inactive for years and then activate when the client’s resistance is lowered, as when a client is being treated for cancer. There’s no such thing as tertiary infection, and superinfection doesn’t apply in this case. 12. A client has active TB. Which of the following symptoms will he exhibit? 1. Chest and lower back pain 2. Chills, fever, night sweats, and hemoptysis Ans-17. 1. The client is showing s/s of active TB and, because of the productive cough, is highly contagious. He should be admitted to the hospital, placed in respiratory isolation, and three sputum cultures should be obtained to confirm the diagnosis. He would most likely be given isoniazid and two or three other antitubercular antibiotics until the diagnosis is confirmed, then isolation and treatment would continue if the cultures were positive for TB. After 7 to 10 days, three more consecutive sputum cultures will be obtained. If they’re negative, he would be considered non-contagious and may be sent home, although he’ll continue to take the antitubercular drugs for 9 to 12 months. 18. A client is diagnosed with active TB and started on triple antibiotic therapy. What signs and symptoms would the client show if therapy is inadequate? 1. Decreased shortness of breath 2. Improved chest x-ray 3. Nonproductive cough 4. Positive acid-fast bacilli in a sputum sample after 2 months of treatment. Ans-18. 4. Continuing to have acid-fast bacilli in the sputum after 2 months indicated continued infection. 19. A client diagnosed with active TB would be hospitalized primarily for which of the following reasons? 1. To evaluate his condition 2. To determine his compliance 3. to prevent spread of the disease 4. To determine the need for antibiotic therapy. Ans-19. 3. The client with active TB is highly contagious until three consecutive sputum cultures are negative, so he’s put in respiratory isolation in the hospital. 20. A high level of oxygen exerts which of the following effects on the lung? 1. Improves oxygen uptake 2. Increases carbon dioxide levels 3. Stabilizes carbon dioxide levels 4. Reduces amount of functional alveolar surface area Ans-20. 4. Oxygen toxicity causes direct pulmonary trauma, reducing the amount of alveolar surface area available for gaseous exchange, which results in increased carbon dioxide levels and decreased oxygen uptake. 21. A 24-year-old client comes into the clinic complaining of right-sided chest pain and shortness of breath. He reports that it started suddenly. The assessment should include which of the following interventions? 1. Auscultation of breath sounds 2. Chest x-ray 3. Echocardiogram 4. Electrocardiogram (ECG) Ans-21. 1. Because the client is short of breath, listening to breath sounds is a good idea. He may need a chest x-ray and an ECG, but a physician must order these tests. Unless a cardiac source for the client’s pain is identified, he won’t need an echocardiogram. 22. A client with shortness of breath has decreased to absent breath sounds on the right side, from the apex to the base. Which of the following conditions would best explain this? 1. Acute asthma 2. Chronic bronchitis 3. Pneumonia 4. Spontaneous pneumothorax Ans-22. 4. A spontaneous pneumothorax occurs when the client’s lung collapses, causing an acute decrease in the amount of functional lung used in oxygenation. The sudden collapse was the cause of his chest pain and shortness of breath. An asthma attack would show wheezing breath sounds, and bronchitis would have rhonchi. Pneumonia would have bronchial breath sounds over the area of consolidation. 23. Which of the following treatments would the nurse expect for a client with a spontaneous pneumothorax? 1. Antibiotics 2. Bronchodilators 3. Chest tube placement 4. Hyperbaric chamber Ans-23. 3. The only way to re-expand the lung is to place a chest tube on the right side so the air in the pleural space can be removed and the lung re-expanded. 24. Which of the following methods is the best way to confirm the diagnosis of a pneumothorax? 1. Auscultate breath sounds 2. Have the client use an incentive spirometer 3. Take a chest x-ray 4. stick a needle in the area of decreased breath sounds Ans-24. 3. A chest x-ray will show the area of collapsed lung if pneumothorax is present as well as the volume of air in the pleural space. Listening to breath sounds won’t confirm a diagnosis. An IS is used to encourage deep breathing. A needle thoracotomy is done only in an emergency and only by someone trained to do it. 25. A pulse oximetry gives what type of information about the client? 1. Amount of carbon dioxide in the blood 2. Amount of oxygen in the blood 3. Percentage of hemoglobin carrying oxygen 4. Respiratory rate Ans-25. 3. The pulse oximeter determines the percentage of hemoglobin carrying oxygen. This doesn’t ensure that the oxygen being carried through the bloodstream is actually being taken up by the tissue. 26. What effect does hemoglobin amount have on oxygenation status? 1. No effect 3 2. Reposition the airway. 3. Leave the airway in place until the client gets used to it. 4. Remove the airway and attempt suctioning without it. Ans-31. 1. If a client gags or coughs after nasopharyngeal airway placement, the tube may be too long. The nurse should remove it and insert a shorter one. Simply repositioning the airway won’t solve the problem. The client won’t get used to the tube because it’s the wrong size. Suctioning without a nasopharyngeal airway causes trauma to the natural airway. 32. An 87-year-old client requires long term ventilator therapy. He has a tracheostomy in place and requires frequent suctioning. Which of the following techniques is correct? 1. Using intermittent suction while advancing the catheter. 2. Using continuous suction while withdrawing the catheter. 3. Using intermittent suction while withdrawing the catheter. 4. Using continuous suction while advancing the catheter. Ans-32. Intermittent suction should be applied during catheter withdrawal. To prevent hypoxia, suctioning shouldn’t last more than 10-seconds at a time. Suction shouldn’t be applied while the catheter is being advanced. 33. A client’s ABG analysis reveals a pH of 7.18, PaCO2 of 72 mm Hg, PaO2 of 77 mm Hg, and HCO – of 24 mEq/L. What do these values indicate? 1. Metabolic acidosis 2. Respiratory alkalosis 3. Metabolic alkalosis 4. Respiratory acidosis Ans-33. 4. 34. A police officer brings in a homeless client to the ER. A chest x-ray suggests he has TB. The physician orders an intradermal injection of 5 tuberculin units/0.1 ml of tuberculin purified derivative. Which needle is appropriate for this injection? 1. 5/8” to ½” 25G to 27G needle. 2. 1” to 3” 20G to 25G needle. 3. ½” to 3/8” 26 or 27G needle. 4. 1” 20G needle. Ans-34. 3. Intradermal injections like those used in TN skin tests are administered in small volumes (usually 0.5 ml or less) into the outer skin layers to produce a local effect. A TB syringe with a ½” to 3/8” 26G or 27G needle should be inserted about 1/8” below the epidermis. 35. A 76-year old client is admitted for elective knee surgery. Physical examination reveals shallow respirations but no signs of respiratory distress. Which of the following is a normal physiologic change related to aging? 1. Increased elastic recoil of the lungs 2. Increased number of functional capillaries in the alveoli 3. Decreased residual volume 4. Decreased vital capacity. Ans-35. 4. Reduction in VC is a normal physiologic change in the older adult. Other normal physiologic changes include decreased elastic recoil of the lungs, fewer functional capillaries in the alveoli, and an increase is residual volume. 36. A 79-year-old client is admitted with pneumonia. Which nursing diagnosis should take priority? 1. Acute pain related to lung expansion secondary to lung infection 2. Risk for imbalanced fluid volume related to increased insensible fluid losses secondary to fever. 3. Anxiety related to dyspnea and chest pain. 4. Ineffective airway clearance related to retained secretions. Ans-36. 4. Pneumonia is an acute infection of the lung parenchyma. The inflammatory reaction may cause an outpouring of exudate into the alveolar spaces, leading to an ineffective airway clearance related to retained secretions. 37. A community health nurse is conducting an educational session with community members regarding TB. The nurse tells the group that one of the first symptoms associated with TB is: 1. A bloody, productive cough 2. A cough with the expectoration of mucoid sputum 3. Chest pain 4. Dyspnea Ans-37. 2. One of the first pulmonary symptoms includes a slight cough with the expectoration of mucoid sputum. 38. A nurse evaluates the blood theophylline level of a client receiving aminophylline (theophylline) by intravenous infusion. The nurse would determine that a therapeutic blood level exists if which of the following were noted in the laboratory report? 1. 5 mcg/mL 2. 15 mcg/mL 3. 25 mcg/mL 4. 30 mcg/mL Ans-38. 2. The therapeutic theophylline blood level range from 10-20 mcg/mL. 39. Isoniazid (INH) and rifampin (Rifadin) have been prescribed for a client with TB. A nurse reviews the medical record of the client. Which of the following, if noted in the client’s history, would require physician notification? 1. Heart disease 2. Allergy to penicillin 3. Hepatitis B 4. Rheumatic fever Ans-39. 3. Isoniazid and rafampin are contraindicated in clients with acute liver disease or a history of hepatic injury. 2. Nasotracheal suctioning to clear secretions 3. Frequent linen changes 4. Frequent offering of a bedpan. Ans-45. 3. Frequent linen changes are appropriate for this client because of diaphoresis. Diaphoresis produces general discomfort. The client should be kept dry to promote comfort. Position changes need to be done every 2 hours. Nasotracheal suctioning is not indicated with the client’s productive cough. Frequent offering of a bedpan is not indicated by the data provided in this scenario. 46. The cyanosis that accompanies bacterial pneumonia is primarily caused by which of the following? 1. Decreased cardiac output 2. Pleural effusion 3. Inadequate peripheral circulation 4. Decreased oxygenation of the blood. Ans-46. 4. A client with pneumonia has less lung surface available for the diffusion of gases because of the inflammatory pulmonary response that creates lung exudate and results in reduced oxygenation of the blood. The client becomes cyanotic because blood is not adequately oxygenated in the lungs before it enters the peripheral circulation. 47. Which of the following mental status changes may occur when a client with pneumonia is first experiencing hypoxia? 1. Coma 2. Apathy 3. Irritability 4. Depression Ans-47. 3. Clients who are experiencing hypoxia characteristically exhibit irritability, restlessness, or anxiety as initial mental status changes. As the hypoxia becomes more pronounced, the client may become confused and combative. Coma is a late clinical manifestation of hypoxia. Apathy and depression are not symptoms of hypoxia. 48. A client with pneumonia has a temperature ranging between 101* and 102*F and periods of diaphoresis. Based on this information, which of the following nursing interventions would be a priority? 1. Maintain complete bed rest 2. Administer oxygen therapy 3. Provide frequent linen changes. 4. Provide fluid intake of 3 L/day Ans-48. 4. A fluid intake of at least 3 L/day should be provided to replace any fluid loss occurring as a result the fever and diaphoresis; this is a high-priority intervention. 49. Which of the following would be an appropriate expected outcome for an elderly client recovering from bacterial pneumonia? 1. A respiratory rate of 25 to 30 breaths per minute 2. The ability to perform ADL’s without dyspnea 3. A maximum loss of 5 to 10 pounds of body weight 4. Chest pain that is minimized by splinting the ribcage. Ans-49. 2. An expected outcome for a client recovering from pneumonia would be the ability to perform ADL’s without experiencing dyspnea. A respiratory rate of 25 to 30 breaths/minute indicates the client is experiencing tachypnea, which would not be expected on recovery. A weight loss of 5-10 pounds is undesirable; the expected outcome would be to maintain normal weight. A client who is recovering from pneumonia should experience decreased or no chest pain. 50. Which of the following symptoms is common in clients with TB? 1. Weight loss 2. Increased appetite 3. Dyspnea on exertion 4. Mental status changes Ans-50. 1. TB typically produces anorexia and weight loss. Other signs and symptoms may include fatigue, low-grade fever, and night sweats. 51. The nurse obtains a sputum specimen from a client with suspected TB for laboratory study. Which of the following laboratory techniques is most commonly used to identify tubercle bacilli in sputum? 1. Acid-fast staining 2. Sensitivity testing 3. Agglutination testing 4. Dark-field illumination Ans-51. 1. The most commonly used technique to identify tubercle bacilli is acid-fast staining. The bacilli have a waxy surface, which makes them difficult to stain in the lab. However, once they are stained, the stain is resistant to removal, even with acids. Therefore, tubercle bacilli are often called acid-fast bacilli. 52. Which of the following antituberculus drugs can cause damage to the eighth cranial nerve? 1. Streptomycin 2. Isoniazid 3. Para-aminosalicylic acid 4. Ethambutol hydrochloride Ans-52. 1. Streptomycin is an aminoglycoside, and eight cranial nerve damage (ototoxicity) is a common side effect from aminoglycosides. 53. The client experiencing eighth cranial nerve damage will most likely report which of the following symptoms? 1. Vertigo 2. Facial paralysis 3. Impaired vision 4. Difficulty swallowing 2. Visit the clinic weekly to ask him whether he is taking his medications regularly. 3. Notify the physician of the client’s non-compliance and request a different prescription. 4. Remind the client that TB can be fatal if not taken properly. Ans-59. 1. Directly observed therapy (DOT) can be implemented with clients who are not compliant with drug therapy. In DOT, a responsible person, who may be a family member or a health care provider, observes the client taking the medication. Visiting the client, changing the prescription, or threatening the client will not ensure compliance if the client will not or cannot follow the prescribed treatment. Blood Disorders 1. The nurse is preparing to teach a client with microcytic hypochromic anemia about the diet to follow after discharge. Which of the following foods should be included in the diet? A. Eggs B. Lettuce C. Citrus fruits D. Cheese Ans-1. 1. One of the microcytic, hypochromic anemia’s is iron-deficiency amenia. A rich source of iron is needed in the diet, and eggs are high in iron. Other foods high in iron include organ and muscle (dark) meats; shellfish, shrimp, and tuna; enriched, whole-grain, and fortified cereals and breads; legumes, nuts, dried fruits, and beans; oatmeal; and sweet potatoes. Dark green leafy vegetables and citrus fruits are good sources of vitamin C. Cheese is a good source of calcium. 2. The nurse would instruct the client to eat which of the following foods to obtain the best supply of vitamin B12? 1. Whole grains 2. Green leafy vegetables 3. Meats and dairy products 4. Broccoli and Brussels sprouts Ans-2. 3. Good sources of vitamin B12 include meats and dairy products. Whole grains are a good source of thiamine. Green leafy vegetables are good sources of niacin, folate, and carotenoids (precursors of vitamin A). Broccoli and Brussels sprouts are good sources of ascorbic acid (vitamin C). 3. The nurse has just admitted a 35-year-old female client who has a serum B12 concentration of 800 pg/ml. Which of the following laboratory findings would cue the nurse to focus the client history on specific drug or alcohol abuse? 1. Total bilirubin, 0.3 mg/dL 2. Serum creatinine, 0.5 mg/dL 3. Hemoglobin, 16 g/dL 4. Folate, 1.5 ng/mL Ans-3. 4. The normal range of folic acid is 1.8 to 9 ng/mL, and the normal range of vitamin B12 is 200 to 900 pg/mL. A low folic acid level in the presence of a normal vitamin B12 level is indicative of a primary folic acid-deficiency anemia. Factors that affect the absorption of folic acid are drugs such as methotrexate, oral contraceptives, antiseizure drugs, and alcohol. The total bilirubin, serum creatinine, and hemoglobin values are within normal limits. 4. The nurse understands that the client with pernicious anemia will have which distinguishing laboratory findings? 1. Schilling’s test, elevated 2. Intrinsic factor, absent. 3. Sedimentation rate, 16 mm/hour 4. RBCs 5.0 million Ans-4. 2. The defining characteristic of pernicious anemia, a megaloblastic anemia, is lack of the intrinsic factor, which results from atrophy of the stomach wall. Without the intrinsic factor, vitamin B12 cannot be absorbed in the small intestines, and folic acid needs vitamin B12 for DNA synthesis of RBCs. The gastric analysis was done to determine the primary cause of the anemia. An elevated excretion of the injected radioactive vitamin B12, which is protocol for the first and second stage of the Schilling test, indicates that the client has the intrinsic factor and can absorb vitamin B12 into the intestinal tract. A sedimentation rate of 16 mm/hour is normal for both men and women and is a nonspecific test to detect the presence of inflammation. It is not specific to anemias. An RBC value of 5.0 million is a normal value for both men and women and does not indicate anemia. 5. The nurse devises a teaching plan for the patient with aplastic anemia. Which of the following is the most important concept to teach for health maintenance? 1. Eat animal protein and dark leafy vegetables each day 2. Avoid exposure to others with acute infection 3. Practice yoga and meditation to decrease stress and anxiety 4. Get 8 hours of sleep at night and take naps during the day Ans-5. 2. Clients with aplastic anemia are severely immunocompromised and at risk for infection and possible death related to bone marrow suppression and pancytopenia. Strict aseptic technique and reverse isolation are important measures to prevent infection. Although diet, reduced stress, and rest are valued in supporting health, the potentially fatal consequence of an acute infection places it as a priority for teaching the client about health maintenance. Animal meat and dark green leafy vegetables, good sources of vitamin B12 and folic acid, should be included in the daily diet. Yoga and meditation are good complimentary therapies to reduce stress. Eight hours of rest and naps are good for spacing and pacing activity and rest. 6. A client comes into the health clinic 3 years after undergoing a resection of the terminal ileum complaining of weakness, shortness of breath, and a sore tongue. Which client statement indicates a need for intervention and client teaching? 1. “I have been drinking plenty of fluids.” 2. “I have been gargling with warm salt water for my sore tongue.” 3. “I have 3 to 4 loose stools per day.” 4. “I take a vitamin B12 tablet every day.” Ans-6. 4. Vitamin B12 combines with intrinsic factor in the stomach and is then carried to the ileum, where it is absorbed in the bloodstream. In this situation, vitamin B12 cannot be absorbed regardless of the amount of oral intake of sources of vitamin B12 such as animal protein or vitamin B12 tablets. Vitamin B12 need to be injected every month, because the ileum has been 11. A client with pernicious anemia asks why she must take vitamin B12 injections for the rest of her life. What is the nurse’s best response? 1. “The reason for your vitamin deficiency is an inability to absorb the vitamin because the stomach is not producing sufficient acid.” 2. “The reason for your vitamin deficiency is an inability to absorb the vitamin because the stomach is not producing sufficient intrinsic factor.” 3. “The reason for your vitamin deficiency is an excessive excretion of the vitamin because of kidney dysfunction.” 4. “The reason for your vitamin deficiency is an increased requirement for the vitamin because of rapid red blood cell production.” Ans-11. 2. Most clients with pernicious anemia have deficient production of intrinsic factor in the stomach. Intrinsic factor attaches to the vitamin in the stomach and forms a complex that allows the vitamin to be absorbed in the small intestine. The stomach is producing enough acid, there is not an excessive excretion of the vitamin, and there is not a rapid production of RBCs in this condition. 12. The nurse is assessing a client’s activity intolerance by having the client walk on a treadmill for 5 minutes. Which of the following indicates an abnormal response? 1. Pulse rate increased by 20 bpm immediately after the activity 2. Respiratory rate decreased by 5 breaths/minute 3. Diastolic blood pressure increased by 7 mm Hg 4. Pulse rate within 6 bpm of resting phase after 3 minutes of rest. Ans-12. 2. The normal physiologic response to activity is an increased metabolic rate over the resting basal rate. The decrease in respiratory rate indicates that the client is not strong enough to complete the mechanical cycle of respiration needed for gas exchange. The post activity pulse is expected to increase immediately after activity but by no more than 50 bpm if it is strenuous activity. The diastolic blood pressure is expected to rise but by no more than 15 mm Hg. The pulse returns to within 6 bpm of the resting pulse after 3 minutes of rest. 13. When comparing the hematocrit levels of a post-op client, the nurse notes that the hematocrit decreased from 36% to 34% on the third day even though the RBC and hemoglobin values remained stable at 4.5 million and 11.9 g/dL, respectively. Which nursing intervention is most appropriate? 1. Check the dressing and drains for frank bleeding 2. Call the physician 3. Continue to monitor vital signs 4. Start oxygen at 2L/min per NC Ans-13. 3. The nurse should continue to monitor the client, because this value reflects a normal physiologic response. The physician does not need to be called, and oxygen does not need to be started based on these laboratory findings. Immediately after surgery, the client’s hematocrit reflects a falsely high value related to the body’s compensatory response to the stress of sudden loss of fluids and blood. Activation of the intrinsic pathway and the renin-angiotensin cycle via antidiuretic hormone produces vasoconstriction and retention of fluid for the first 1 to 2 day post- op. By the second to third day, this response decreases and the client’s hematocrit level is more reflective of the amount of RBCs in the plasma. Fresh bleeding is a less likely occurrence on the third post-op day but is not impossible; however, the nurse would have expected to see a decrease in the RBC and hemoglobin values accompanying the hematocrit. 14. A client is to receive epoetin (Epogen) injections. What laboratory value should the nurse assess before giving the injection? 1. Hematocrit 2. Partial thromboplastin time 3. Hemoglobin concentration 4. Prothrombin time Ans-14. 1. Epogen is a recombinant DNA form of erythropoietin, which stimulates the production of RBCs and therefore causes the hematocrit to rise. The elevation in hematocrit causes an elevation in blood pressure; therefore, the blood pressure is a vital sign that should be checked. The PTT, hemoglobin level, and PT are not monitored for this drug. 15. A client states that she is afraid of receiving vitamin B12 injections because of the potential toxic reactions. What is the nurse’s best response to relieve these fears? 1. “Vitamin B12 will cause ringing in the eats before a toxic level is reached.” 2. “Vitamin B12 may cause a very mild skin rash initially.” 3. “Vitamin B12 may cause mild nausea but nothing toxic.” 4. “Vitamin B12 is generally free of toxicity because it is water soluble.” Ans-15. 4. Vitamin B12 is a water-soluble vitamin. When water-soluble vitamins are taken in excess of the body’s needs, they are filtered through the kidneys and excreted. Vitamin B12is considered to be nontoxic. Adverse reactions that have occurred are believed to be related to impurities or to the preservative in B12 preparations. Ringing in the ears, skin rash, and nausea are not considered to be related to vitamin B12 administration. 16. A client with microcytic anemia is having trouble selecting food items from the hospital menu. Which food is best for the nurse to suggest for satisfying the client’s nutritional needs and personal preferences? 1. Egg yolks 2. Brown rice 3. Vegetables 4. Tea Ans-16. 2. Brown rice is a source of iron from plant sources (nonheme iron). Other sources of nonheme iron are whole-grain cereals and breads, dark green vegetables, legumes, nuts, dried fruits (apricots, raisins, and dates), oatmeal, and sweet potatoes. Egg yolks have iron but it is not as well absorbed as iron from other sources. Vegetables are a good source of vitamins that may facilitate iron absorption. Tea contains tannin, which combines with nonheme iron, preventing its absorption. 17. A client with macrocytic anemia has a burn on her foot and states that she had been watching television while lying on a heating pad. What is the nurse’s first response? 1. Assess for potential abuse 2. Check for diminished sensations 3. Document the findings 4. Clean and dress the area Ans-17. 2. Macrocytic anemias can result from deficiencies in vitamin B12 or ascorbic acid. Only vitamin B12 deficiency causes diminished sensations of peripheral nerve endings. The nurse Ans-22. 3. Anemia stems from a decreased number of red blood cells and the resulting deficiency in oxygen and body tissues. Clotting factors, such as factor VIII, relate to the body’s ability to form blood clots and aren’t related to anemia, not is carbon dioxide of T antibodies. 23. Which of the following cells is the precursor to the red blood cell (RBC)? 1. B cell 2. Macrophage 3. Stem cell 4. T cell Ans-23. 3. The precursor to the RBC is the stem cell. B cells, macrophages, and T cells and lymphocytes, not RBC precursors. 24. Which of the following symptoms is expected with hemoglobin of 10 g/dl? 1. None 2. Pallor 3. Palpitations 4. Shortness of breath Ans-24. 1. Mild anemia usually has no clinical signs. Palpitations, SOB, and pallor are all associated with severe anemia. 25. Which of the following diagnostic findings are most likely for a client with aplastic anemia? 1. Decreased production of T-helper cells 2. Decreased levels of white blood cells, red blood cells, and platelets 3. Increased levels of WBCs, RBCs, and platelets 4. Reed-Sternberg cells and lymph node enlargement Ans-25. 2. In aplastic anemia, the most likely diagnostic findings are decreased levels of all the cellular elements of the blood (pancytopenia). T-helper cell production doesn’t decrease in aplastic anemia. Reed-Sternberg cells and lymph node enlargement occur with Hodgkin’s disease. 26. A client with iron deficiency anemia is scheduled for discharge. Which instruction about prescribed ferrous gluconate therapy should the nurse include in the teaching plan? 1. “Take the medication with an antacid.” 2. “Take the medication with a glass of milk.” 3. “Take the medication with cereal.” 4. “Take the medication on an empty stomach.” Ans-26. 4. Preferably, ferrous gluconate should be taken on an empty stomach. Ferrous gluconae should not be taken with antacids, milk, or whole-grain cereals because these foods reduce iron absorption. 27. Which of the following disorders results from a deficiency of factor VIII? 1. Sickle cell disease 2. Christmas disease 3. Hemophilia A 4. Hemophilia B Ans-27. 3. Hemophilia A results from a deficiency of factor VIII. Sickle cell disease is caused by a defective hemoglobin molecule. Christmas disease, also called hemophilia B, results in a factor IX deficiency. 28. The nurse explains to the parents of a 1-year-old child admitted to the hospital in a sickle cell crisis that the local tissue damage the child has on admission is caused by which of the following? 1. Autoimmune reaction complicated by hypoxia 2. Lack of oxygen in the red blood cells 3. Obstruction to circulation 4. Elevated serum bilirubin concentration. Ans-28. 3. Characteristic sickle cells tend to cause “log jams” in capillaries. This results in poor circulation to local tissues, leading to ischemia and necrosis. The basic defect in sickle cell disease is an abnormality in the structure of RBCs. The erythrocytes are sickle-shaped, rough in texture, and rigid. Sickle cell disease is an inherited disease, not an autoimmune reaction. Elevated serum bilirubin concentrations are associated with jaundice, not sickle cell disease. 29. The mothers asks the nurse why her child’s hemoglobin was normal at birth but now the child has S hemoglobin. Which of the following responses by the nurse is most appropriate? 1. “The placenta bars passage of the hemoglobin S from the mother to the fetus.” 2. “The red bone marrow does not begin to produce hemoglobin S until several months after birth.” 3. “Antibodies transmitted from you to the fetus provide the newborn with temporary immunity.” 4. “The newborn has a high concentration of fetal hemoglobin in the blood for some time after birth.” Ans-29. 4. Sickle cell disease is an inherited disease that is present at birth. However, 60% to 80% of a newborns hemoglobin is fetal hemoglobin, which has a structure different from that of hemoglobin S or hemoglobin A. Sickle cell symptoms usually occur about 4 months after birth, when hemoglobin S begins to replace the fetal hemoglobin. The gene for sickle cell disease is transmitted at the time of conception, not passed through the placenta. Some hemoglobin S is produced by the fetus near term. The fetus produces all its own hemoglobin from the earliest production in the first trimester. Passive immunity conferred by maternal antibodies is not related to sickle cell disease, but this transmission of antibodies is important to protect the infant from various infections during early infancy. 30. Which of the following would the nurse identify as the priority nursing diagnosis during a toddler’s vasoocclusive sickle cell crisis? 1. Ineffective coping related to the presence of a life-threatening disease 2. Decreased cardiac output related to abnormal hemoglobin formation 3. Pain related to tissue anoxia 4. Excess fluid volume related to infection. Ans-30. 3. For the child in a sickle cell crisis, pain is the priority nursing diagnosis because the sickled cells clump and obstruct the blood vessels, leading to occlusive and subsequent tissue ischemia. Although ineffective coping may be important, it is not the priority. Decreased cardiac 36. Because of the risks associated with administration of factor VIII concentrate, the nurse would teach the client’s family to recognize and report which of the following? 1. Yellowing of the skin 2. Constipation 3. Abdominal distention 4. Puffiness around the eyes Ans-36. 1. Because factor VIII concentrate is derived from large pools of human plasma, the risk of hepatitis is always present. Clinical manifestations of hepatitis include yellowing of the skin, mucous membranes, and sclera. Use of factor VIII concentrate is not associated with constipation, abdominal distention, or puffiness around the eyes. 37. A child suspected of having sickle cell disease is seen in a clinic, and laboratory studies are performed. A nurse checks the lab results, knowing that which of the following would be increased in this disease? 1. Platelet count 2. Hematocrit level 3. Reticulocyte count 4. Hemoglobin level Ans-37. 3. A diagnosis is established based on a complete blood count, examination for sickled red blood cells in the peripheral smear, and hemoglobin electrophoresis. Laboratory studies will show decreased hemoglobin and hematocrit levels and a decreased platelet count, and increased reticulocyte count, and the presence of nucleated red blood cells. Increased reticulocyte counts occur in children with sickle cell disease because the life span of their sickled red blood cells is shortened. 38. A clinic nurse instructs the mother of a child with sickle cell disease about the precipitating factors related to pain crisis. Which of the following, if identified by the mother as a precipitating factor, indicates the need for further instructions? 1. Infection 2. Trauma 3. Fluid overload 4. Stress Ans-38. 3. Pain crisis may be precipitated by infection, dehydration, hypoxia, trauma, or physical or emotional stress. The mother of a child with sickle cell disease should encourage fluid intake of 1 ½ to 2 times the daily requirement to prevent dehydration. 39. Laboratory studies are performed for a child suspected of having iron deficiency anemia. The nurse reviews the laboratory results, knowing that which of the following results would indicate this type of anemia? 1. An elevated hemoglobin level 2. A decreased reticulocyte count 3. An elevated RBC count 4. Red blood cells that are microcytic and hypochromic Ans-39. 4. The results of a CBC in children with iron deficiency anemia will show decreased hemoglobin levels and microcytic and hypochromic red blood cells. The red blood cell count is decreased. The reticulocyte count is usually normal or slightly elevated. 40. A pediatric nurse health educator provides a teaching session to the nursing staff regarding hemophilia. Which of the following information regarding this disorder would the nurse plan to include in the discussion? 1. Hemophilia is a Y linked hereditary disorder 2. Males inherit hemophilia from their fathers 3. Females inherit hemophilia from their mothers 4. Hemophilia A results from a deficiency of factor VIII Ans-40. 4. Males inherit hemophilia from their mothers, and females inherit the carrier status from their fathers. Hemophilia is inherited in a recessive manner via a genetic defect on the X- chromosome. Hemophilia A results from a deficiency of factor VIII. Hemophilia B (Christmas disease) is a deficiency of factor IX. Cancer 1. Which of the following conditions is not a complication of Hodgkin’s disease? 1. Anemia 2. Infection 3. Myocardial Infarction 4. Nausea Ans-1. 3. Complications of Hodgkin’s are pancytopenia, nausea, and infection. Cardiac involvement usually doesn’t occur. 2. Which of the following laboratory values is expected for a client just diagnosed with chronic lymphocytic leukemia? 1. Elevated sedimentation rate 2. Uncontrolled proliferation of granulocytes 3. Thrombocytopenia and increased lymphocytes 4. Elevated aspartate aminotransferase and alanine aminotransferase levels. Ans-2. 3. Chronic lymphocytic leukemia shows a proliferation of small abnormal mature B lymphocytes and decreased antibody response. Thrombocytopenia also is often present. Uncontrolled proliferation of granulocytes occurs in myelogenous leukemia. 3. At the time of diagnosis of Hodgkin’s lymphoma, which of the following areas is often involved? 1. Back 2. Chest 3. Groin 4. Neck Ans-3. 4. At the time of diagnosis, a painless cervical lesion is often present. The back, chest, and groin areas aren’t involved. 4. According to a standard staging classification of Hodgkin’s disease, which of the following criteria reflects stage II? 1. Involvement of extra lymphatic organs or tissues 2. Involvement of single lymph node region or structure 3. Involvement of two or more lymph node regions or structures. Ans-10. 1. Calcium is released when the bone is destroyed. This causes an increase in serum calcium levels. MM doesn’t affect potassium, sodium, or magnesium levels. 11. Giving instructions for breast self-examination is particularly important for clients with which of the following medical problems? 1. Cervical dysplasia 2. A dermoid cyst 3. Endometrial polyps 4. Ovarian cancer Ans-11. 4. Clients with ovarian cancer are at increased risk for breast cancer. Breast self- examination supports early detection and treatment and is very important. 12. During a routine physical examination, a firm mass is palpated in the right breast of a 35- year-old woman. Which of the following findings or client history would suggest cancer of the breast as opposed to fibrocystic disease? 1. History of early menarche 2. Cyclic changes in mass size 3. History of an ovulatory cycles 4. Increased vascularity of the breast Ans-12. 4. Increase in breast size or vascularity is consistent with cancer of the breast. Early menarche as well as late menopause or a history of an ovulatory cycles are associated with fibrocystic disease. Masses associated with fibrocystic disease of the breast are firm, most often located in the upper outer quadrant of the breast, and increase in size prior to menstruation. They may be bilateral in a mirror image and are typically well demarcated and freely moveable. 13. The client with which of the following types of lung cancer has the best prognosis? 1. Adenocarcinoma 2. Oat cell 3. Squamous cell 4. Small cell Ans-13. 3. Squamous cell carcinoma is a slow-growing, rarely metastasizing type of cancer. Adenocarcinoma is the next best lung cancer to have in terms of prognosis. Oat cell and small cell carcinoma are the same. Small cell carcinoma grows rapidly and is quick to metastasize. 14. Warning signs and symptoms of lung cancer include persistent cough, bloody sputum, dyspnea, and which of the other following symptoms? 1. Dizziness 2. Generalized weakness 3. Hypotension 4. Recurrent pleural effusion Ans-14. 4. Recurring episodes of pleural effusions can be caused by the tumor and should be investigated. Dizziness, generalized weakness, and hypotension aren’t typically considered warning signals, but may occur in advanced stages of cancer. 15. A centrally located tumor would produce which of the following symptoms? 1. Coughing 2. Hemoptysis 3. Pleuritic pain 4. Shoulder pain Ans-15. 1. Centrally located pulmonary tumors are found in the upper airway (vocal cords) and usually obstruct airflow, producing such symptoms as coughing, wheezing, and stridor. Small cell tumors tend to be located in the lower airways and often cause hemoptysis. As the tumor invades the pleural space, it may cause pleuritic pain. Pancoast tumors that occur in the apices may cause shoulder pain. 16. Which of the following interventions is the key to increasing the survival rates of clients with lung cancer? 1. Early bronchoscopy 2. Early detection 3. High-dose chemotherapy 4. Smoking cessation Ans-16. 2. Early detection of cancer when the cells may be premalignant and potentially curable would be most beneficial. However, a tumor must be 1 cm in diameter before it’s detectable on a chest x-ray, so this is difficult. A bronchoscopy may help identify cell type but may not increase survival rate. High-dose chemotherapy has minimal effect on long-term survival. Smoking cessation won’t reverse the process but may help prevent further decompensation. 17. A client has been diagnosed with lung cancer and requires a wedge resection. How much of the lung is removed? 1. One entire lung 2. A lobe of the lung 3. A small, localized area near the surface of the lung. 4. A segment of the lung, including a bronchiole and its alveoli. Ans-17. 3. A small area of tissue close to the surface of the lung is removed in a wedge resection. An entire lung is removed in a pneumonectomy. A segment of the lung is removed in a segmental resection and a lobe is removed in a lobectomy. 18. When a client has a lobectomy, what fills the space where the lobe was? 1. The space stays empty. 2. The surgeon fills the space with gel 3. The lung space fills up with serous fluid 4. The remaining lobe or lobes overexpand to fill the space. Ans-18. 4. The remaining lobe or lobes over expand slightly to fill the space previously occupied by the removed tissue. The diaphragm is carried higher on the operative side to further reduce the empty space. The space can’t remain “empty” because truly empty would imply a vacuum, which would interfere with the intrathoracic pressure changes that allow breathing. The surgeon doesn’t use a gel to fill the space. Serous fluid overproduction would compress the remaining lobes, diminish their function and possibly, cause a mediastinal shift. 19. Which of the following is the primary goal for surgical resection of lung cancer? 1. To remove the tumor and all surrounding tissue. erythrocytes taking on a spheroid shape and isn’t a feature in leukemia. Mature cells aren’t produced in adequate numbers. Hirsutism and growth delay can be a result of large doses of steroids but isn’t common in leukemia. Anemia, not polycythemia, occurs. Clotting times would be prolonged. 25. A child is seen in the pediatrician’s office for complaints of bone and joint pain. Which of the following other assessment findings may suggest leukemia? 1. Abdominal pain 2. Increased activity level 3. Increased appetite 4. Petechiae Ans-25. 4. The most frequent signs and symptoms of leukemia are a result of infiltration of the bone marrow. These include fever, pallor, fatigue, anorexia, and petechiae, along with bone and joint pain. Increased appetite can occur but it usually isn’t a presenting symptom. Abdominal pain may be caused by areas of inflammation from normal flora within the GI tract or any number of other causes. 26. Which of the following assessment findings in a client with leukemia would indicate that the cancer has invaded the brain? 1. Headache and vomiting. 2. Restlessness and tachycardia 3. Hypervigilant and anxious behavior 4. Increased heart rate and decreased blood pressure. Ans-26. 1. The usual effect of leukemic infiltration of the brain is increased intracranial pressure. The proliferation of cells interferes with the flow of cerebrospinal fluid in the subarachnoid space and at the base of the brain. The increased fluid pressure causes dilation of the ventricles, which creates symptoms of severe headache, vomiting, irritability, lethargy, increased blood pressure, decreased heart rate, and eventually, coma. Often children with a variety of illnesses are hypervigilant and anxious when hospitalized. 27. Which of the following types of leukemia carries the best prognosis? 1. Acute lymphoblastic leukemia 2. Acute myelogenous leukemia 3. Basophilic leukemia 4. Eosinophilia leukemia Ans-27. 1. Acute lymphoblastic leukemia, which accounts for more than 80% of all childhood cases, carries the best prognosis. Acute myelogenous leukemia, with several subtypes, accounts for most of the other leukemias affecting children. Basophilic and eosinophillic leukemia are named for the specific cells involved. These are much rarer and carry a poorer prognosis. 28. Which of the following is the reason to perform a spinal tap on a client newly diagnosed with leukemia? 1. To rule out meningitis 2. To decrease intracranial pressure 3. To aid in classification of the leukemia 4. To assess for central nervous system infiltration Ans-28. 4. A spinal tap is performed to assess for central nervous system infiltration. It wouldn’t be done to decrease ICP nor does it aid in the classification of the leukemia. Spinal taps can result in brain stem herniation in cases of ICP. A spinal tap can be done to rule out meningitis but this isn’t the indication for the test on a leukemic client. 29. Which of the following tests in performed on a client with leukemia before initiation of therapy to evaluate the child’s ability to metabolize chemotherapeutic agents? 1. Lumbar puncture 2. Liver function studies 3. Complete blood count (CBC) 4. Peripheral blood smear Ans-29. 2. Liver and kidney function studies are done before initiation of chemotherapy to evaluate the child’s ability to metabolize the chemotherapeutic agents. A CBC is performed to assess for anemia and white blood cell count. A peripheral blood smear is done to assess the maturity and morphology of red blood cells. A lumbar puncture is performed to assess for central nervous system infiltration. 30. Which of the following immunizations should not be given to a 4-month-old sibling of a client with leukemia? 1. Diphtheria and tetanus and pertussis (DPT) vaccine. 2. Hepatitis B vaccine 3. Haemophilus influenza type b vaccines (Hib) 4. Oral poliovirus vaccine (OPV) Ans-30. 4. OPV is a live attenuated virus excreted in the stool. The excreted virus can be communicated to the immunosuppressed child, resulting in an overwhelming infection. Inactivated polio vaccine would be indicated because it isn’t a live virus and wouldn’t pose the threat of infection. DTP, Hib, and hepatitis B vaccines can be given accordingly to the recommended schedule. 31. Which of the following medications usually is given to a client with leukemia as prophylaxis against P. carinii pneumonia? 1. Bactrim 2. Oral nystatin suspension 3. Prednisone 4. Vincristine (Oncovin) Ans-31. 1. The most frequent cause of death from leukemia is overwhelming infection. P. carinii infection is lethal to a child with leukemia. As prophylaxis against P. cariniipneumonia, continuous low doses of co-trimoxazole (Bactrim) are frequently prescribed. Oral nystatin suspension would be indicated for the treatment of thrush. Prednisone isn’t an antibiotic and increases susceptibility to infection. Vincristine is an antineoplastic agent. 32. In which of the following diseases would bone marrow transplantation not be indicated in a newly diagnosed client? 1. Acute lymphocytic leukemia 2. Chronic myeloid leukemia 3. Severe aplastic anemia 4. Severe combined immunodeficiency Ans-37. 3. Viruses may be one of multiple agents acting to initiate carcinogenesis and have been associated with several types of cancer. Increased stress has been associated with causing the growth and proliferation of cancer cells. Two forms of radiation, ultraviolet and ionizing, can lead to cancer. A diet high in fat may be a factor in the development of breast, colon, and prostate cancers. High-fiber diets may reduce the risk of colon cancer. 38. The client with cancer is receiving chemotherapy and develops thrombocytopenia. The nurse identifies which intervention as the highest priority in the nursing plan of care? 1. Ambulation three times a day 2. Monitoring temperature 3. Monitoring the platelet count 4. Monitoring for pathological factors Ans-38. 3. Thrombocytopenia indicates a decrease in the number of platelets in the circulating blood. A major concern is monitoring for and preventing bleeding. Option 2 relates to monitoring for infection particularly if leukopenia is present. Options 1 and 4, although important in the plan of care are not related directly to thrombocytopenia. 39. A client is diagnosed with multiple myeloma. The client asks the nurse about the diagnosis. The nurse bases the response on which of the following descriptions of this disorder? 1. Malignant exacerbation in the number of leukocytes. 2. Altered red blood cell production. 3. Altered production of lymph nodes 4. Malignant proliferation of plasma cells and tumors within the bone. 39. A client is diagnosed with multiple myeloma. The client asks the nurse about the diagnosis. The nurse bases the response on which of the following descriptions of this disorder? 1. Malignant exacerbation in the number of leukocytes. 2. Altered red blood cell production. 3. Altered production of lymph nodes 4. Malignant proliferation of plasma cells and tumors within the bone. Ans-39. 4. Multiple myeloma is a B cell neoplastic condition characterized by abnormal malignant proliferation of plasma cells and the accumulation of mature plasma cells in the bone marrow. Option 1 describes the leukemic process. Options 2 and 3 are not characteristics of multiple myeloma. 40. The nurse is reviewing the laboratory results of a client diagnosed with multiple myeloma. Which of the following would the nurse expect to note specifically in this disorder? 1. Decreased number of plasma cells in the bone marrow. 2. Increased WBC’s 3. Increased calcium levels 4. Decreased blood urea nitrogen Ans-40. 3. Findings indicative of multiple myeloma are an increased number of plasma cells in the bone marrow, anemia, hypercalcemia caused by the release of calcium from the deteriorating bone tissue, and an elevated blood urea nitrogen level. An increased white blood cell count may or may not be present and is not related specifically to multiple myeloma. 41. The nurse is developing a plan of care for the client with multiple myeloma. The nurse includes which priority intervention in the plan of care? 1. Coughing and deep breathing 2. Encouraging fluids 3. Monitoring red blood cell count 4. Providing frequent oral care Ans-41. 2. Hypercalcemia caused by bone destruction is a priority concern in the client with multiple myeloma. The nurse should administer fluids in adequate amounts to maintain and output of 1.5 to 2 L a day. Clients require about 3 L of fluid pre day. The fluid is needed not only to dilute the calcium overload but also to prevent protein from precipitating in renal tubules. Options 1, 3, and 4 may be components in the plan of care but are not the priority in this client. 42. The oncology nurse specialist provides an educational session to nursing staff regarding the characteristics of Hodgkin’s disease. The nurse determines that further education is needed if a nursing staff member states that which of the following is characteristic of the disease? 1. Presence of Reed-Sternberg cells 2. Involvement of lymph nodes, spleen, and liver 3. Occurs most often in the older client 4. Prognosis depends on the stage of the disease Ans-42. 3. Hodgkin’s disease is a disorder of young adults. Options 1, 2, and 4 are characteristics of this disease. 43. The nurse is reviewing the laboratory results of a client receiving chemotherapy. The platelet count is 10,000 cells/mm. Based on this laboratory value, the priority nursing assessment is which of the following? 1. Assess level of consciousness 2. Assess temperature 3. Assess bowel sounds 4. Assess skin turgor Ans-43. 1. A high risk of hemorrhage exists when the platelet count is fewer than 20,000. Fatal central nervous system hemorrhage or massive gastrointestinal hemorrhage can occur when the platelet count is fewer than 10,000. The client should be assessed for changes in levels of consciousness, which may be an early indication of an intracranial hemorrhage. Option 2 is a priority nursing assessment when the white blood cell count is low and the client is at risk for an infection. 44. The nurse is caring for a client following a modified radical mastectomy. Which assessment finding would indicate that the client is experiencing a complication related to this surgery? 1. Sanguineous drainage in the Jackson-Pratt drain 2. Pain at the incisional site 3. Complaints of decreased sensation near the operative site 3. Administer a cytotoxic agent to keep the regimen on schedule even if blood return is not present. 4. If unable to aspirate blood, reposition the client and encourage the client to cough. 5. Contact the health care provider about verifying placement if the status is questionable. Ans-49. 1, 2, 4, 5. A major concern with intravenous administration of cytotoxic agents is vessel irritation or extravasation. The Oncology Nursing Society and hospital guidelines require frequent evaluation of blood return when administering vesicant or nonvesicant chemotherapy due to the risk of extravasation. These guidelines apply to peripheral and central venous lines. In addition, central venous lines may be long-term venous access devices. Thus, difficulty drawing or aspirating blood may indicate the line is against the vessel wall or may indicate the line has occlusion. Having the client cough or move position may change the status of the line if it is temporarily against a vessel wall. Occlusion warrants more thorough evaluation via x-ray study to verify placement if the status is questionable and may require a declotting regimen. 50. A client with stomach cancer is admitted to the oncology unit after vomiting for 3 days. Physical assessment findings include irregular pulse, muscle twitching, and complaints of prickling sensations in the fingers and hands. Laboratory results include a potassium level of 2.9 mEq/L, a pH of 7.46, and a bicarbonate level of 29 mEq/L. The client is experiencing: 1. Respiratory alkalosis 2. Respiratory acidosis 3. Metabolic alkalosis 4. Metabolic acidosis Ans-50. 3. The client is experiencing metabolic alkalosis caused by loss of hydrogen and chloride ions from excessive vomiting. This is shown by a pH of 7.46 and elevated bicarbonate level of 29 mEq/L. 51. A 32-year-old woman meets with the nurse on her first office visit since undergoing a left mastectomy. When asked how she is doing, the woman states her appetite is still not good, she is not getting much sleep because she doesn’t go to bed until her husband is asleep, and she is really anxious to get back to work. Which of the following nursing interventions should the nurse explore to support the client’s current needs? 1. Call the physician to discuss allowing the client to return to work earlier. 2. Suggest that the client learn relaxation techniques to help with her insomnia 3. Perform a nutritional assessment to assess for anorexia 4. Ask open-ended questions about sexuality issues related to her mastectomy Ans-51. 4. The content of the client’s comments suggests that she is avoiding intimacy with her husband by waiting until he is asleep before going to bed. Addressing sexuality issues is appropriate for a client who has undergone a mastectomy. Rushing her return to work may debilitate her and add to her exhaustion. Suggesting that she learn relaxation techniques to help her with her insomnia is appropriate; however, the nurse must first address the psychosocial and sexual issues that are contributing to her sleeping difficulties. A nutritional assessment may be useful, but there is no indication that she has anorexia. 52. One of the most serious blood coagulation complications for individuals with cancer and for those undergoing cancer treatments is disseminated intravascular coagulation (DIC). The most common cause of this bleeding disorder is: 1. Underlying liver disease 2. Brain metastasis 3. Intravenous heparin therapy 4. Sepsis Ans-52. 4. Bacterial endotoxins released from gram-negative bacteria activate the Hageman factor or coagulation factor XII. This factor inhibits coagulation via the intrinsic pathway of homeostasis, as well as stimulating fibrinolysis. Liver disease can cause multiple bleeding abnormalities resulting in chronic, subclinic DIC; however, sepsis is the most common cause. 53. A pneumonectomy is a surgical procedure sometimes indicated for treatment of non- small-cell lung cancer. A pneumonectomy involves removal of: 1. An entire lung field 2. A small, wedge-shaped lung surface 3. One lobe of a lung 4. One or more segments of a lung lobe Ans-53. 1. A pneumonectomy is the removal of an entire lung field. A wedge resection refers to removal of a wedge-shaped section of lung tissue. A lobectomy is the removal of one lobe. Removal of one or more segments of a lung lobe is called a partial lobectomy. 54. A 36-year-old man with lymphoma presents with signs of impending septic shock 9 days after chemotherapy. The nurse could expect which of the following to be present? 1. Flushing, decreased oxygen saturation, mild hypotension 2. Low-grade fever, chills, tachycardia 3. Elevated temperature, oliguria, hypotension 4. High-grade fever, normal blood pressure, increased respirations Ans-54. 2. Nine days after chemotherapy, one would expect the client to be immunocompromised. The clinical signs of shock reflect changes in cardiac function, vascular resistance, cellular metabolism, and capillary permeability. Low-grade fever, tachycardia, and flushing may be early signs of shock. The client with impending signs of septic shock may not have decreased oxygen saturation levels. Oliguria and hypotension are late signs of shock. Urine output can be initially normal or increased. 55. Which of the following represents the most appropriate nursing intervention for a client with pruritis caused by cancer or the treatments? 1. Administration of antihistamines 2. Steroids 3. Silk sheets 4. Medicated cool baths Ans-55. 4. Nursing interventions to decrease the discomfort of pruritus include those that prevent vasodilation, decrease anxiety, and maintain skin integrity and hydration. Medicated baths with salicyclic acid or colloidal oatmeal can be soothing as a temporary relief. The use of antihistamines or topical steroids depends on the cause of pruritus, and these agents should be used with caution. Using silk sheets is not a practical intervention for the hospitalized client with pruritis.
Docsity logo



Copyright © 2024 Ladybird Srl - Via Leonardo da Vinci 16, 10126, Torino, Italy - VAT 10816460017 - All rights reserved